LSAT and Law School Admissions Forum

Get expert LSAT preparation and law school admissions advice from PowerScore Test Preparation.

 Administrator
PowerScore Staff
  • PowerScore Staff
  • Posts: 8917
  • Joined: Feb 02, 2011
|
#41632
Complete Question Explanation
(The complete setup for this game can be found here: lsat/viewtopic.php?t=2813)

The correct answer choice is (E)

If N is presented first, we need to ensure that K gives the second, fourth, and sixth presentations in accordance with the first rule. And, since neither T nor S can be given sixth, it follows that P must be the sixth presentation given, an inference that eliminates answer choices (A) and (B):
PT65_D11 LG Explanations_game_#2_#8_diagram 1.png
Of course, a Templates-based approach would obviate the need for a local diagram, because only Template 4 would allow N to be presented first.

Answer choice (A) is incorrect, because the subject of the sixth presentation must be P, not O.

Answer choice (B) is incorrect, because P must be presented sixth, not second.

Answer choice (C) is incorrect, because S is presented by K, and must therefore be presented either second or fourth.

Answer choice (D) is incorrect, because T is presented by K, and must therefore be presented either second or fourth.

Answer choice (E) is the correct answer choice, because W is presented either third or fifth.
You do not have the required permissions to view the files attached to this post.

Get the most out of your LSAT Prep Plus subscription.

Analyze and track your performance with our Testing and Analytics Package.